Prove that $O(3^{2n}) subseteq O(2^{3n})$












-1












$begingroup$


I need to prove that $O(3^{2n}) subseteq O(2^{3n})$. So far I have made this solution:



I)Lets assume that this is true, and that there $ exists space c in mathbb{R}^+ $ such as $$3^{2n} leq 2^{3n} * c$$



II) therefore this fraction must be greater or equal as $1$



$$frac{c*2^{3n}}{3^{2n}} geq 1$$



III) lets look on $$lim_{ntoinfty} frac{c*2^{3n}}{3^{2n}}=(frac{8}{9})^n=0$$ since the limit is equal to $0$ there occurs an conflict that shows us that II) can not ever be greater or equal as $1$. Is my solution correct? Any help is appreciated.










share|cite|improve this question











$endgroup$












  • $begingroup$
    I'm pretty sure this is false.
    $endgroup$
    – Lucas Henrique
    Dec 16 '18 at 22:19










  • $begingroup$
    The statement in your question's title is the other way round to the statement in the body.
    $endgroup$
    – Patrick Stevens
    Dec 16 '18 at 22:25










  • $begingroup$
    Edited the title, thanks
    $endgroup$
    – nocturne
    Dec 16 '18 at 22:53






  • 1




    $begingroup$
    Now the title and the body are different (small-o vs big-O) and both wrong.
    $endgroup$
    – Did
    Dec 17 '18 at 0:04










  • $begingroup$
    Edited again, it should be fine now.
    $endgroup$
    – nocturne
    Dec 17 '18 at 8:44
















-1












$begingroup$


I need to prove that $O(3^{2n}) subseteq O(2^{3n})$. So far I have made this solution:



I)Lets assume that this is true, and that there $ exists space c in mathbb{R}^+ $ such as $$3^{2n} leq 2^{3n} * c$$



II) therefore this fraction must be greater or equal as $1$



$$frac{c*2^{3n}}{3^{2n}} geq 1$$



III) lets look on $$lim_{ntoinfty} frac{c*2^{3n}}{3^{2n}}=(frac{8}{9})^n=0$$ since the limit is equal to $0$ there occurs an conflict that shows us that II) can not ever be greater or equal as $1$. Is my solution correct? Any help is appreciated.










share|cite|improve this question











$endgroup$












  • $begingroup$
    I'm pretty sure this is false.
    $endgroup$
    – Lucas Henrique
    Dec 16 '18 at 22:19










  • $begingroup$
    The statement in your question's title is the other way round to the statement in the body.
    $endgroup$
    – Patrick Stevens
    Dec 16 '18 at 22:25










  • $begingroup$
    Edited the title, thanks
    $endgroup$
    – nocturne
    Dec 16 '18 at 22:53






  • 1




    $begingroup$
    Now the title and the body are different (small-o vs big-O) and both wrong.
    $endgroup$
    – Did
    Dec 17 '18 at 0:04










  • $begingroup$
    Edited again, it should be fine now.
    $endgroup$
    – nocturne
    Dec 17 '18 at 8:44














-1












-1








-1





$begingroup$


I need to prove that $O(3^{2n}) subseteq O(2^{3n})$. So far I have made this solution:



I)Lets assume that this is true, and that there $ exists space c in mathbb{R}^+ $ such as $$3^{2n} leq 2^{3n} * c$$



II) therefore this fraction must be greater or equal as $1$



$$frac{c*2^{3n}}{3^{2n}} geq 1$$



III) lets look on $$lim_{ntoinfty} frac{c*2^{3n}}{3^{2n}}=(frac{8}{9})^n=0$$ since the limit is equal to $0$ there occurs an conflict that shows us that II) can not ever be greater or equal as $1$. Is my solution correct? Any help is appreciated.










share|cite|improve this question











$endgroup$




I need to prove that $O(3^{2n}) subseteq O(2^{3n})$. So far I have made this solution:



I)Lets assume that this is true, and that there $ exists space c in mathbb{R}^+ $ such as $$3^{2n} leq 2^{3n} * c$$



II) therefore this fraction must be greater or equal as $1$



$$frac{c*2^{3n}}{3^{2n}} geq 1$$



III) lets look on $$lim_{ntoinfty} frac{c*2^{3n}}{3^{2n}}=(frac{8}{9})^n=0$$ since the limit is equal to $0$ there occurs an conflict that shows us that II) can not ever be greater or equal as $1$. Is my solution correct? Any help is appreciated.







computational-complexity






share|cite|improve this question















share|cite|improve this question













share|cite|improve this question




share|cite|improve this question








edited Dec 17 '18 at 8:44







nocturne

















asked Dec 16 '18 at 20:06









nocturnenocturne

689




689












  • $begingroup$
    I'm pretty sure this is false.
    $endgroup$
    – Lucas Henrique
    Dec 16 '18 at 22:19










  • $begingroup$
    The statement in your question's title is the other way round to the statement in the body.
    $endgroup$
    – Patrick Stevens
    Dec 16 '18 at 22:25










  • $begingroup$
    Edited the title, thanks
    $endgroup$
    – nocturne
    Dec 16 '18 at 22:53






  • 1




    $begingroup$
    Now the title and the body are different (small-o vs big-O) and both wrong.
    $endgroup$
    – Did
    Dec 17 '18 at 0:04










  • $begingroup$
    Edited again, it should be fine now.
    $endgroup$
    – nocturne
    Dec 17 '18 at 8:44


















  • $begingroup$
    I'm pretty sure this is false.
    $endgroup$
    – Lucas Henrique
    Dec 16 '18 at 22:19










  • $begingroup$
    The statement in your question's title is the other way round to the statement in the body.
    $endgroup$
    – Patrick Stevens
    Dec 16 '18 at 22:25










  • $begingroup$
    Edited the title, thanks
    $endgroup$
    – nocturne
    Dec 16 '18 at 22:53






  • 1




    $begingroup$
    Now the title and the body are different (small-o vs big-O) and both wrong.
    $endgroup$
    – Did
    Dec 17 '18 at 0:04










  • $begingroup$
    Edited again, it should be fine now.
    $endgroup$
    – nocturne
    Dec 17 '18 at 8:44
















$begingroup$
I'm pretty sure this is false.
$endgroup$
– Lucas Henrique
Dec 16 '18 at 22:19




$begingroup$
I'm pretty sure this is false.
$endgroup$
– Lucas Henrique
Dec 16 '18 at 22:19












$begingroup$
The statement in your question's title is the other way round to the statement in the body.
$endgroup$
– Patrick Stevens
Dec 16 '18 at 22:25




$begingroup$
The statement in your question's title is the other way round to the statement in the body.
$endgroup$
– Patrick Stevens
Dec 16 '18 at 22:25












$begingroup$
Edited the title, thanks
$endgroup$
– nocturne
Dec 16 '18 at 22:53




$begingroup$
Edited the title, thanks
$endgroup$
– nocturne
Dec 16 '18 at 22:53




1




1




$begingroup$
Now the title and the body are different (small-o vs big-O) and both wrong.
$endgroup$
– Did
Dec 17 '18 at 0:04




$begingroup$
Now the title and the body are different (small-o vs big-O) and both wrong.
$endgroup$
– Did
Dec 17 '18 at 0:04












$begingroup$
Edited again, it should be fine now.
$endgroup$
– nocturne
Dec 17 '18 at 8:44




$begingroup$
Edited again, it should be fine now.
$endgroup$
– nocturne
Dec 17 '18 at 8:44










2 Answers
2






active

oldest

votes


















1












$begingroup$

This is false.



Suppose it holds: then, since obviously $3^{2n} in O(3^{2n}) subseteq O(2^{3n}) implies 3^{2n} in O(2^{3n})$, $lim_{n to infty}, sup |frac{9}{8}|^n < infty$, which obviously does not hold.






share|cite|improve this answer









$endgroup$





















    1












    $begingroup$

    More generally,
    when is
    $a^{bn} subseteq c^{dn}$?



    $a^{bn}
    =(e^{ln(a)})^{bn}
    =e^{ln(a)b n}
    =(e^n)^{ln(a)b }
    =(e^n)^{ln(a^b) }
    $
    .



    Therefore
    $a^{bn} subseteq c^{dn}
    iff ln(a^b) le ln(c^d)
    iff a^b le c^d
    $
    .



    Putting
    $a=3, b=2, c=2, d=3$,
    this is true
    iff
    $3^2 le 2^3$,
    which is false.



    For the case
    $c=b, d=a$,
    i.e.,
    when is
    $a^{bn} subseteq b^{an}
    $
    ,
    this is the old problem
    of when
    $a^b le b^a$,
    or
    $a^{1/a} le b^{1/b}$.



    It is well known that
    this is true when
    $e le b lt a$.



    The case
    $a < e < b$
    is trickier.
    I have a result that shows
    this is decided if
    $ab > e^2$
    but I can't find it right now.
    I'll add to this if
    I find it.






    share|cite|improve this answer









    $endgroup$













      Your Answer





      StackExchange.ifUsing("editor", function () {
      return StackExchange.using("mathjaxEditing", function () {
      StackExchange.MarkdownEditor.creationCallbacks.add(function (editor, postfix) {
      StackExchange.mathjaxEditing.prepareWmdForMathJax(editor, postfix, [["$", "$"], ["\\(","\\)"]]);
      });
      });
      }, "mathjax-editing");

      StackExchange.ready(function() {
      var channelOptions = {
      tags: "".split(" "),
      id: "69"
      };
      initTagRenderer("".split(" "), "".split(" "), channelOptions);

      StackExchange.using("externalEditor", function() {
      // Have to fire editor after snippets, if snippets enabled
      if (StackExchange.settings.snippets.snippetsEnabled) {
      StackExchange.using("snippets", function() {
      createEditor();
      });
      }
      else {
      createEditor();
      }
      });

      function createEditor() {
      StackExchange.prepareEditor({
      heartbeatType: 'answer',
      autoActivateHeartbeat: false,
      convertImagesToLinks: true,
      noModals: true,
      showLowRepImageUploadWarning: true,
      reputationToPostImages: 10,
      bindNavPrevention: true,
      postfix: "",
      imageUploader: {
      brandingHtml: "Powered by u003ca class="icon-imgur-white" href="https://imgur.com/"u003eu003c/au003e",
      contentPolicyHtml: "User contributions licensed under u003ca href="https://creativecommons.org/licenses/by-sa/3.0/"u003ecc by-sa 3.0 with attribution requiredu003c/au003e u003ca href="https://stackoverflow.com/legal/content-policy"u003e(content policy)u003c/au003e",
      allowUrls: true
      },
      noCode: true, onDemand: true,
      discardSelector: ".discard-answer"
      ,immediatelyShowMarkdownHelp:true
      });


      }
      });














      draft saved

      draft discarded


















      StackExchange.ready(
      function () {
      StackExchange.openid.initPostLogin('.new-post-login', 'https%3a%2f%2fmath.stackexchange.com%2fquestions%2f3043089%2fprove-that-o32n-subseteq-o23n%23new-answer', 'question_page');
      }
      );

      Post as a guest















      Required, but never shown

























      2 Answers
      2






      active

      oldest

      votes








      2 Answers
      2






      active

      oldest

      votes









      active

      oldest

      votes






      active

      oldest

      votes









      1












      $begingroup$

      This is false.



      Suppose it holds: then, since obviously $3^{2n} in O(3^{2n}) subseteq O(2^{3n}) implies 3^{2n} in O(2^{3n})$, $lim_{n to infty}, sup |frac{9}{8}|^n < infty$, which obviously does not hold.






      share|cite|improve this answer









      $endgroup$


















        1












        $begingroup$

        This is false.



        Suppose it holds: then, since obviously $3^{2n} in O(3^{2n}) subseteq O(2^{3n}) implies 3^{2n} in O(2^{3n})$, $lim_{n to infty}, sup |frac{9}{8}|^n < infty$, which obviously does not hold.






        share|cite|improve this answer









        $endgroup$
















          1












          1








          1





          $begingroup$

          This is false.



          Suppose it holds: then, since obviously $3^{2n} in O(3^{2n}) subseteq O(2^{3n}) implies 3^{2n} in O(2^{3n})$, $lim_{n to infty}, sup |frac{9}{8}|^n < infty$, which obviously does not hold.






          share|cite|improve this answer









          $endgroup$



          This is false.



          Suppose it holds: then, since obviously $3^{2n} in O(3^{2n}) subseteq O(2^{3n}) implies 3^{2n} in O(2^{3n})$, $lim_{n to infty}, sup |frac{9}{8}|^n < infty$, which obviously does not hold.







          share|cite|improve this answer












          share|cite|improve this answer



          share|cite|improve this answer










          answered Dec 16 '18 at 22:24









          Lucas HenriqueLucas Henrique

          1,059414




          1,059414























              1












              $begingroup$

              More generally,
              when is
              $a^{bn} subseteq c^{dn}$?



              $a^{bn}
              =(e^{ln(a)})^{bn}
              =e^{ln(a)b n}
              =(e^n)^{ln(a)b }
              =(e^n)^{ln(a^b) }
              $
              .



              Therefore
              $a^{bn} subseteq c^{dn}
              iff ln(a^b) le ln(c^d)
              iff a^b le c^d
              $
              .



              Putting
              $a=3, b=2, c=2, d=3$,
              this is true
              iff
              $3^2 le 2^3$,
              which is false.



              For the case
              $c=b, d=a$,
              i.e.,
              when is
              $a^{bn} subseteq b^{an}
              $
              ,
              this is the old problem
              of when
              $a^b le b^a$,
              or
              $a^{1/a} le b^{1/b}$.



              It is well known that
              this is true when
              $e le b lt a$.



              The case
              $a < e < b$
              is trickier.
              I have a result that shows
              this is decided if
              $ab > e^2$
              but I can't find it right now.
              I'll add to this if
              I find it.






              share|cite|improve this answer









              $endgroup$


















                1












                $begingroup$

                More generally,
                when is
                $a^{bn} subseteq c^{dn}$?



                $a^{bn}
                =(e^{ln(a)})^{bn}
                =e^{ln(a)b n}
                =(e^n)^{ln(a)b }
                =(e^n)^{ln(a^b) }
                $
                .



                Therefore
                $a^{bn} subseteq c^{dn}
                iff ln(a^b) le ln(c^d)
                iff a^b le c^d
                $
                .



                Putting
                $a=3, b=2, c=2, d=3$,
                this is true
                iff
                $3^2 le 2^3$,
                which is false.



                For the case
                $c=b, d=a$,
                i.e.,
                when is
                $a^{bn} subseteq b^{an}
                $
                ,
                this is the old problem
                of when
                $a^b le b^a$,
                or
                $a^{1/a} le b^{1/b}$.



                It is well known that
                this is true when
                $e le b lt a$.



                The case
                $a < e < b$
                is trickier.
                I have a result that shows
                this is decided if
                $ab > e^2$
                but I can't find it right now.
                I'll add to this if
                I find it.






                share|cite|improve this answer









                $endgroup$
















                  1












                  1








                  1





                  $begingroup$

                  More generally,
                  when is
                  $a^{bn} subseteq c^{dn}$?



                  $a^{bn}
                  =(e^{ln(a)})^{bn}
                  =e^{ln(a)b n}
                  =(e^n)^{ln(a)b }
                  =(e^n)^{ln(a^b) }
                  $
                  .



                  Therefore
                  $a^{bn} subseteq c^{dn}
                  iff ln(a^b) le ln(c^d)
                  iff a^b le c^d
                  $
                  .



                  Putting
                  $a=3, b=2, c=2, d=3$,
                  this is true
                  iff
                  $3^2 le 2^3$,
                  which is false.



                  For the case
                  $c=b, d=a$,
                  i.e.,
                  when is
                  $a^{bn} subseteq b^{an}
                  $
                  ,
                  this is the old problem
                  of when
                  $a^b le b^a$,
                  or
                  $a^{1/a} le b^{1/b}$.



                  It is well known that
                  this is true when
                  $e le b lt a$.



                  The case
                  $a < e < b$
                  is trickier.
                  I have a result that shows
                  this is decided if
                  $ab > e^2$
                  but I can't find it right now.
                  I'll add to this if
                  I find it.






                  share|cite|improve this answer









                  $endgroup$



                  More generally,
                  when is
                  $a^{bn} subseteq c^{dn}$?



                  $a^{bn}
                  =(e^{ln(a)})^{bn}
                  =e^{ln(a)b n}
                  =(e^n)^{ln(a)b }
                  =(e^n)^{ln(a^b) }
                  $
                  .



                  Therefore
                  $a^{bn} subseteq c^{dn}
                  iff ln(a^b) le ln(c^d)
                  iff a^b le c^d
                  $
                  .



                  Putting
                  $a=3, b=2, c=2, d=3$,
                  this is true
                  iff
                  $3^2 le 2^3$,
                  which is false.



                  For the case
                  $c=b, d=a$,
                  i.e.,
                  when is
                  $a^{bn} subseteq b^{an}
                  $
                  ,
                  this is the old problem
                  of when
                  $a^b le b^a$,
                  or
                  $a^{1/a} le b^{1/b}$.



                  It is well known that
                  this is true when
                  $e le b lt a$.



                  The case
                  $a < e < b$
                  is trickier.
                  I have a result that shows
                  this is decided if
                  $ab > e^2$
                  but I can't find it right now.
                  I'll add to this if
                  I find it.







                  share|cite|improve this answer












                  share|cite|improve this answer



                  share|cite|improve this answer










                  answered Dec 17 '18 at 0:17









                  marty cohenmarty cohen

                  73.6k549128




                  73.6k549128






























                      draft saved

                      draft discarded




















































                      Thanks for contributing an answer to Mathematics Stack Exchange!


                      • Please be sure to answer the question. Provide details and share your research!

                      But avoid



                      • Asking for help, clarification, or responding to other answers.

                      • Making statements based on opinion; back them up with references or personal experience.


                      Use MathJax to format equations. MathJax reference.


                      To learn more, see our tips on writing great answers.




                      draft saved


                      draft discarded














                      StackExchange.ready(
                      function () {
                      StackExchange.openid.initPostLogin('.new-post-login', 'https%3a%2f%2fmath.stackexchange.com%2fquestions%2f3043089%2fprove-that-o32n-subseteq-o23n%23new-answer', 'question_page');
                      }
                      );

                      Post as a guest















                      Required, but never shown





















































                      Required, but never shown














                      Required, but never shown












                      Required, but never shown







                      Required, but never shown

































                      Required, but never shown














                      Required, but never shown












                      Required, but never shown







                      Required, but never shown







                      Popular posts from this blog

                      Quarter-circle Tiles

                      build a pushdown automaton that recognizes the reverse language of a given pushdown automaton?

                      Mont Emei